LSAT and Law School Admissions Forum

Get expert LSAT preparation and law school admissions advice from PowerScore Test Preparation.

User avatar
 Dave Killoran
PowerScore Staff
  • PowerScore Staff
  • Posts: 5862
  • Joined: Mar 25, 2011
|
#92233
Complete Question Explanation
(The complete setup for this game can be found here: lsat/viewtopic.php?f=371&t=15040)

The correct answer choice is (E).

This is another Hurdle the Uncertainty question. If L is assigned to two committees, the following results:

G2-Q11-d1.png

However, from the second rule, M must be assigned to a committee, and so M must be assigned to the Quality Committee. G cannot be on either the Quality or Sales Committee due to the fourth rule, and so the officer spaces must be occupied by F or H:

G2-Q11-d2.png

Accordingly, answer choice (E) is correct.

Again, note that this hypothetical is sufficient to prove question #10, answer choice (E) correct.
You do not have the required permissions to view the files attached to this post.

Get the most out of your LSAT Prep Plus subscription.

Analyze and track your performance with our Testing and Analytics Package.